Difference between revisions of "1977 AHSME Problems/Problem 9"
(→Solution) |
m (→Problem 9) |
||
Line 30: | Line 30: | ||
− | + | ==Solution== | |
Solution by e_power_pi_times_i | Solution by e_power_pi_times_i | ||
If arcs <math>AB</math>, <math>BC</math>, and <math>CD</math> are congruent, then <math>\measuredangle ACB = \measuredangle BDC = \measuredangle CBD = \theta</math>. Because <math>ABCD</math> is cyclic, <math>\measuredangle CAD = \measuredangle CBD = \theta</math>, and <math>\measuredangle ADB = \measuredangle ACB = \theta</math>. Then, <math>\measuredangle EAD = \measuredangle EDA = \dfrac{180^\circ - 40^\circ}{2} = 70^\circ</math>. <math>\theta = 55^\circ</math>. <math>\measuredangle ACD = 180^\circ - 55^\circ - 110^\circ = \boxed{\textbf{(B) }15^\circ}</math>. | If arcs <math>AB</math>, <math>BC</math>, and <math>CD</math> are congruent, then <math>\measuredangle ACB = \measuredangle BDC = \measuredangle CBD = \theta</math>. Because <math>ABCD</math> is cyclic, <math>\measuredangle CAD = \measuredangle CBD = \theta</math>, and <math>\measuredangle ADB = \measuredangle ACB = \theta</math>. Then, <math>\measuredangle EAD = \measuredangle EDA = \dfrac{180^\circ - 40^\circ}{2} = 70^\circ</math>. <math>\theta = 55^\circ</math>. <math>\measuredangle ACD = 180^\circ - 55^\circ - 110^\circ = \boxed{\textbf{(B) }15^\circ}</math>. |
Latest revision as of 11:30, 21 November 2016
Problem 9
In the adjoining figure and arc , arc , and arc all have equal length. Find the measure of .
Solution
Solution by e_power_pi_times_i
If arcs , , and are congruent, then . Because is cyclic, , and . Then, . . .